The Student Room Group

Proof, modular arithmetic, probs & stats

Hey, too many questions.

1.Is the reasoning in the first pic ok? Edit. No it wasn't.

2. Is this correct?

3. How is integral of Mx(t) not negative?

4. Is my venn ok? I can get P(C n D) but then am not so sure what is going on, tried a few things. I should be aiming for the space to be 1 right?

5. What is that

Φ1\displaystyle \Phi^{-1}

symbol about? How can I find these values using the z tables?
(edited 11 years ago)

Scroll to see replies

Reply 1
No, the reasoning is a bit faulty - I think you've mixed up where the "not" goes. You can tell it's faulty because it's true that if mn is even then m is even or n is even.
You wanted "mn even => m even or n even" has contrapositive "m odd and n odd => mn odd".
If you're asked to prove that mn is even implies m even or n is even, I think you need a special case of Bézout's Theorem,which states that if hcf(m,n) = k then am+bn = k has solutions. This adapts by:
If m is not even, then hcf(2, m) = 1 and hence am + 2b = 1 has solutions, and hence amn + 2bn = n has solutions. But mn is assumed to be even, and hence amn is, so the LHS is even, and hence the RHS is even, so n is even. Hence at least one of m and n is even.
Your answer on slide 2 is fine.
Mx(t) is not negative because the integrand is positive (exponentials are positive).
Your Venn diagram is fine. P(C U D) = P(we get one of b,e,d,f); you can look at this as P(we don't get a or c).
The inverse-phi is exactly the inverse function of the phi function; that is, if phi(0) = 0.5, then phi^-1(0.5) = 0. You use the phi tables "in reverse" - that is, instead of looking at the row, and following along the columns to reach the right entry, you look for the entry in the table, then find which row/column it lies in.
(edited 11 years ago)
Original post by SubAtomic
Hey, too many questions.

5. What is that

Φ1\displaystyle \Phi^{-1}

symbol about? How can I find these values using the z tables?


It's the inverse of the cumulative normal distribution.

You can either, for standard values, look it up in the small table that usually follows the standard normal distribution table,

or, look for the value you're interested in in the body of the normal distribution table, and see what z values gives that.
Reply 3
Original post by Smaug123
No, the reasoning is a bit faulty - I think you've mixed up where the "not" goes. You can tell it's faulty because it's true that if mn is even then m is even or n is even.


Lol, after a second look it is indeed faulty, that pic is from a while ago, :colondollar:


Original post by Smaug123
You wanted "mn even => m even or n even" has contrapositive "m odd and n odd => mn odd".
If you're asked to prove that mn is even implies m even or n is even, I think you need a special case of Bézout's Theorem,which states that if hcf(m,n) = k then am+bn = k has solutions. This adapts by:
If m is not even, then hcf(2, m) = 1 and hence am + 2b = 1 has solutions, and hence amn + 2bn = n has solutions. But mn is assumed to be even, and hence amn is, so the LHS is even, and hence the RHS is even, so n is even. Hence at least one of m and n is even.
Your answer on slide 2 is fine.


:cool:

Original post by Smaug123
Mx(t) is not negative because the integrand is positive (exponentials are positive).


Hmmm. I thought it should be lambda over t - lambda?

Original post by Smaug123
Your Venn diagram is fine. P(C U D) = P(we get one of b,e,d,f); you can look at this as P(we don't get a or c).


So am I assuming that all events are contained in the sets A B C and D? P(C u D) = 0.5 ? Or am I making false assumptions that the sample space = 1? Maybe I have overcomplicated things :rolleyes:

Original post by Smaug123
The inverse-phi is exactly the inverse function of the phi function; that is, if phi(0) = 0.5, then phi^-1(0.5) = 0. You use the phi tables "in reverse" - that is, instead of looking at the row, and following along the columns to reach the right entry, you look for the entry in the table, then find which row/column it lies in.


:cool:
(edited 11 years ago)
Reply 4
Original post by SubAtomic

Hmmm. I thought it should be lambda over t - lambda?

Ah, I misunderstood you; I thought you meant Mx(t) was negative, which in hindsight was a silly thing for me to think given that you wrote "the integral of Mx(t)" :P I suspect it's only defined for t>lambda, but I have forgotten the circumstances as the picture seems to have vanished!

Original post by SubAtomic

So am I assuming that all events are contained in the sets A B C and D? P(C u D) = 0.5 ? Or am I making false assumptions that the sample space = 1? Maybe I have overcomplicated things :rolleyes:

Yep, the fact that it's a sample space means every possible outcome is in the sample space, so there are six possible outcomes and their total probability is 1. C U D together rules out four of the six outcomes (if the outcome were e, then the outcome is in C U D, etc). To check, I made P(C) = 3/10. We can see that all possible outcomes are contained somewhere in A, B, C and D, because every one of the six possible letters is used somewhere.
(edited 11 years ago)
Reply 5
Original post by Smaug123
Ah, I misunderstood you; I thought you meant Mx(t) was negative, which in hindsight was a silly thing for me to think given that you wrote "the integral of Mx(t)" :P I suspect it's only defined for t>lambda, but I have forgotten the circumstances as the picture seems to have vanished!


I edited my embarrassing proof out and must have taken that pic down, it is back now :smile:


Original post by Smaug123
Yep, the fact that it's a sample space means every possible outcome is in the sample space, so there are six possible outcomes and their total probability is 1. C U D together rules out four of the six outcomes (if the outcome were e, then the outcome is in C U D, etc). To check, I made P(C) = 3/10. We can see that all possible outcomes are contained somewhere in A, B, C and D, because every one of the six possible letters is used somewhere.


Yep, I was just making it awkward for myself for some reason :cool:
Reply 6
Is this right?

3333725mod 11\displaystyle 3^{3^{3^{3}}}\equiv -72 \equiv 5\mod \ 11
Reply 7
Original post by ghostwalker
It's the inverse of the cumulative normal distribution.

You can either, for standard values, look it up in the small table that usually follows the standard normal distribution table,

or, look for the value you're interested in in the body of the normal distribution table, and see what z values gives that.


Hey, am still uncertain, oh, think it just clicked, so I was supposed to be looking for the value that gives 0.45 as the rhs of the distribution is full? I was looking at 0.475 :rolleyes:
Reply 8
Original post by SubAtomic
I edited my embarrassing proof out and must have taken that pic down, it is back now :smile:

Yep, I was right the first time - it says that it's only defined for t<lambda. It's like cos1(t)\cos^{-1}(t) is only defined for 1t1-1 \le t \le 1.
Reply 9
Original post by SubAtomic
Is this right?

3333725mod 11\displaystyle 3^{3^{3^{3}}}\equiv -72 \equiv 5\mod \ 11
I don't think so.

3^3 = 27

3^3^3 = 3^27 = 7625597484987

So you are after 3^7625597484987

Use FLT to make this less scary...
Original post by SubAtomic
Hey, am still uncertain, oh, think it just clicked, so I was supposed to be looking for the value that gives 0.45 as the rhs of the distribution is full? I was looking at 0.475 :rolleyes:


OK.

It's more usual for tables to give the cumulative distribution from -infinity, rather than from 0, as the values will differ by 0.5 in the body of the table.

As long as you're happy that such things are taken care of if/as necessary in your calculations.
Reply 11
D Franklin and ghostwalker in one thread :adore:

So the mod one I must have done, not sure what I did looking at it now :s-smilie:

So would the correct solution be

37625597484987(310)x3713941089mod11\displaystyle 3^{7625597484987} \equiv (3^{10})^x \cdot 3^7 \equiv 1 \cdot 3 \cdot 9 \cdot 4 \equiv 108 \equiv 9 \mod 11

Maybe?
(edited 11 years ago)
Reply 12
Original post by ghostwalker
OK.

It's more usual for tables to give the cumulative distribution from -infinity, rather than from 0, as the values will differ by 0.5 in the body of the table.

As long as you're happy that such things are taken care of if/as necessary in your calculations.


Thanks, I am not sure if I will ever be totally happy with my calculations, but I am as happy as I can be, thanks for the help :cool:
Original post by SubAtomic
D Franklin and ghostwalker in one thread :adore:


:rofl: I'll do the easy bit!
Reply 14
Original post by ghostwalker
:rofl: I'll do the easy bit!


:biggrin: all my questions are easy for you :tongue: it is probably me that causes the most difficulty haha.
Reply 15
Original post by SubAtomic

3. How is integral of Mx(t) not negative?


What restrictions do you have on t? Look at the form of Mx(t)M_x(t) given to you after its been integrated. What do you notice?
Reply 16
Original post by shamika
What restrictions do you have on t? Look at the form of Mx(t)M_x(t) given to you after its been integrated. What do you notice?


t < lambda

Confused, I will show what am doing then maybe it will be something ridiculous as usual :rolleyes:

λe(tλ)x dx=λe(tλ)xtλ\displaystyle \lambda \int e^{(t- \lambda)x} \ dx = \frac{\lambda \cdot e^{(t- \lambda)x}}{t- \lambda}

Where do I go from here to get that equation in the pic? Or is it just something I am supposed to take as given?
(edited 11 years ago)
Reply 17
Original post by SubAtomic
t < lambda

Confused, I will show what am doing then maybe it will be something ridiculous as usual :rolleyes:

λe(tλ)x dx=λe(tλ)xtλ\displaystyle \lambda \int e^{(t- \lambda)x} \ dx = \frac{\lambda \cdot e^{(t- \lambda)x}}{t- \lambda}

Where do I go from here to get that equation in the pic? Or is it just something I am supposed to take as given?


Evaluate it with the limits and then hopefully it'll make sense :wink:
Reply 18
Original post by shamika
Evaluate it with the limits and then hopefully it'll make sense :wink:


:emo: had to look at evaluating integrals with infinity, so I get this

Unparseable latex formula:

\displaystyle \frac{\lambda \cdot e^{-\infty}}{t-\lambda}- \frac{\lambda}{t- \lambda}}



=λeλtλ\displaystyle = \frac{\lambda \cdot e^{-\infty} - \lambda}{t- \lambda}

Is this right? If so I think I may just about get it :redface:
(edited 11 years ago)
Reply 19
Original post by SubAtomic
:emo: had to look at evaluating integrals with infinity, so I get this

Unparseable latex formula:

\displaystyle \frac{\lambda \cdot e^{-\infty}}{t-\lambda}- \frac{\lambda}{t- \lambda}}



=λeλtλ\displaystyle = \frac{\lambda \cdot e^{-\infty} - \lambda}{t- \lambda}

Is this right? If so I think I may just about get it :redface:

Pretty much, yep - the infinite integral should technically be evaluated as a limit, but this works fine if you just use lim(e^-x) = 0 as x->inf.

Quick Reply

Latest